PRACTICE TEST 2 - PRACTICE TESTS WITH DETAILED ANSWER KEYS - SAT Test Prep

SAT Test Prep

CHAPTER 16
PRACTICE TESTS WITH DETAILED ANSWER KEYS

PRACTICE TEST 2

ANSWER SHEET

Directions for Test

• Remove these answer sheets from the book and use them to record your answers to this test.

• This test will require 3 hours and 20 minutes to complete. Take this test in one sitting.

• The time allotment for each section is written clearly at the beginning of each section. This test contains six 25-minute sections, two 20-minute sections, and one 10-minute section.

• This test is 25 minutes shorter than the actual SAT, which will include a 25-minute “experimental” section that does not count toward your score. That section has been omitted from this test.

• You may take one short break during the test, of no more than 10 minutes in length.

• You may only work on one section at any given time.

• You must stop ALL work on a section when time is called.

• If you finish a section before the time has elapsed, check your work on that section. You may NOT work on any other section.

• Do not waste time on questions that seem too difficult for you.

• Use the test book for scratchwork, but you will receive credit only for answers that are marked on the answer sheets.

• You will receive one point for every correct answer.

• You will receive no points for an omitted question.

• For each wrong answer on any multiple-choice question, your score will be reduced by ¼ point.

• For each wrong answer on any “numerical grid-in” question, you will receive no deduction.

When you take the real SAT, you will be asked to fill in your personal information in grids as shown below.

Start with number 1 for each new section. If a section has fewer questions than answer spaces, leave the extra answer spaces blank. Be sure to erase any errors or stray marks completely.

CAUTION

Use the answer spaces in the grids below for Section 2 or Section 3 only if you are told to do so in your test book.

Student-Produced Responses

Start with number 1 for each new section. If a section has fewer questions than answer spaces, leave the extra answer spaces blank. Be sure to erase any errors or stray marks completely.

CAUTION

Use the answer spaces in the grids below for Section 4 or Section 5 only if you are told to do so in your test book.

Student-Produced Responses

Start with number 1 for each new section. If a section has fewer questions than answer spaces, leave the extra answer spaces blank. Be sure to erase any errors or stray marks completely.

CAUTION

Use the answer spaces in the grids below for Section 6 or Section 7 only if you are told to do so in your test book.

Student-Produced Responses

Start with number 1 for each new section. If a section has fewer questions than answer spaces, leave the extra answer spaces blank. Be sure to erase any errors or stray marks completely.

ESSAY
Time—25 minutes


Write your essay on separate sheets of standard lined paper.


The essay gives you an opportunity to show how effectively you can develop and express ideas. You should therefore take care to develop your point of view, present your ideas logically and clearly, and use language precisely.

Your essay must be written on the lines provided on your answer sheet—you will receive no other paper on which to write. You will have enough space if you write on every line, avoid wide margins, and keep your handwriting to a reasonable size. Remember that people who are not familiar with your handwriting will read what you write. Try to write or print so that what you are writing is legible to those readers.

Important reminders:

A pencil is required for the essay. An essay written in ink will receive a score of zero.

Do not write your essay in your test book. You will receive credit only for what you write on your answer sheet.

An off-topic essay will receive a score of zero.

You have twenty-five minutes to write an essay on the topic assigned below.


Consider carefully the issue discussed in the following passage, then write an essay that answers the question posed in the assignment.


The best leaders are not those who seek power or have great political skill. Great leaders—and these are exceptionally rare, especially today—represent the best selves of the people they represent.


Assignment: What are the most important qualities of a leader? Write an essay in which you answer this question and discuss your point of view on this issue. Support your position logically with examples from literature, the arts, history, politics, science and technology, current events, or your experience or observation.


If you finish before time is called, you may check your work on this section only. Do not turn to any other section in the test.

SECTION 2
Time—25 minutes
24 questions


Turn to Section 2 of your answer sheet to answer the questions in this section.



Directions: For each question in this section, select the best answer from among the choices given and fill in the corresponding circle on the answer sheet.



Each sentence below has one or two blanks, each blank indicating that something has been omitted. Beneath the sentence are five words or sets of words labeled A through E. Choose the word or set of words that, when inserted in the sentence, best fits the meaning of the sentence as a whole.

EXAMPLE:

Rather than accepting the theory unquestioningly, Deborah regarded it with -----.

(A) mirth

(B) sadness

(C) responsibility

(D) ignorance

(E) skepticism


1. Even though Alisha had every reason to hold a grudge, she felt that ------- was not a healthful emotion.

(A) resentment

(B) fortitude

(C) sarcasm

(D) elation

(E) fondness

2. Those who expected the governor to be inarticulate were surprised by his -------.

(A) intolerance

(B) fatigue

(C) eloquence

(D) endurance

(E) violence

3. Before the Realist movement, novelists rarely utilized the ------- language of commoners, preferring the more ------- parlance of the upper classes.

(A) normal . . ordinary

(B) elite . . fancy

(C) sympathetic . . wasteful

(D) colloquial . . refined

(E) effective . . utilitarian

4. Many college students are attracted to the ------- life of a journalist; the prospect of exploring the world is very appealing, even if the pay is not.

(A) peripatetic

(B) conventional

(C) tolerant

(D) coordinated

(E) remunerative

5. A position that requires public speaking would be very difficult for one as ------- as he.

(A) vivacious

(B) garrulous

(C) amiable

(D) decent

(E) reticent

6. One example of a ------- relationship is provided by the tickbird, which gets protection and a free meal of ticks from the hippopotamus and in turn supplies free pest removal services.

(A) competitive

(B) deteriorating

(C) symbiotic

(D) regressive

(E) vacillating

7. Early philosophers used ------- alone to reach their conclusions; unlike modern scientists, they did not value the ------- information that comes only from close observation and experimentation.

(A) reason . . empirical

(B) coercion . . mathematical

(C) deduction . . clerical

(D) computation . . intuitive

(E) compassion . . numerical

8. The ------- of many media companies under a single owner is troublesome to those who believe that ------- is essential to the fair and balanced presentation of the news.

(A) retraction . . differentiation

(B) consolidation . . independence

(C) collaboration . . sharing

(D) unification . . dissemination

(E) disintegration . . variety


The following passages are followed by questions based on their content. Answer the questions on the basis of what is stated or implied in the passage and in any introductory material that may be provided.


Questions 9–12 are based on the following passages.

9. Unlike Passage 1, Passage 2 focuses on the distinction between

(A) educating the poor and educating the wealthy

(B) power and knowledge

(C) teachers and students

(D) educated people and uneducated people

(E) schooling and education

First passage: Horace Mann, The Case for Public Schools, a report to the Massachusetts Board of Education in 1848.

Second passage: Printed with the permission of its author, Christopher Black, and College Hill Coaching. © 2005

10. Passage 1 mentions each of the following as benefits of public education to the poor EXCEPT

(A) the diminishment of social distinctions

(B) the improvement of living standards

(C) better ability to counteract greed

(D) increased self-sufficiency

(E) the reduction of crime

11. Passage 1 suggests that the obliteration of “factitious distinctions” (lines 13–14) requires

(A) unlimited access to education

(B) a rigorous curriculum in civics

(C) hostility toward the rich

(D) dedicated teachers

(E) aggressive legislation

12. The author of Passage 2 characterizes the “struggle” (line 21) as

(A) regretful

(B) empowering

(C) illusionary

(D) unwinnable

(E) foreign

Questions 13–18 are based on the following passage.

The following is an essay from a textbook on the history of philosophy published in 1999.

Excerpted from “The Renaissance Interlude,” in Socrates to Sartre, by Samuel Enoch Stumpf, McGraw-Hill, New York, 1999. Reproduced with permission of The McGraw-Hill Companies.

13. Which of the following is the best title for this passage?

(A) The Beginnings of the Scientific Method

(B) Scientific Instruments of the Renaissance

(C) The Art and Science of the Renaissance

(D) Biblical Influence on the Scientific Mode of Thought

(E) The Importance of Hypotheses in Scientific Thinking

14. As it is used in line 8, “stress” most nearly means

(A) anxiety

(B) pressure

(C) emphasis

(D) desperation

(E) contortion

15. It can be inferred from the passage that if pre-Renaissance scientists observed the motions of heavenly bodies, they did so most likely in order to

(A) confirm the formulas that describe the motions of the planets and stars

(B) distinguish the motions of various planets

(C) validate what the Bible says about those bodies

(D) demonstrate the utility of their newly invented instruments

(E) refute the hypotheses of their rival scientists

16. The passage indicates that Galileo did which of the following?

I. invented an important optical instrument

II. instructed another famous scientist

III. made an important astronomical discovery

(A) II only

(B) III only

(C) I and II only

(D) I and III only

(E) I, II, and III

17. The passage indicates that, unlike the “earlier moralists” (line 74), Renaissance scientists began to perceive human behavior as

(A) a matter of free choice

(B) influenced by heavenly bodies

(C) controlled by a metaphysical spirit

(D) affected by animalistic impulses

(E) subject to the laws of physical motion

18. The primary function of the last paragraph is to

(A) propose a solution to a problem

(B) identify those responsible for a discovery

(C) discuss the effects of a change

(D) refute a misconception

(E) address an objection to the author”s thesis

Questions 19–24 are based on the following passage.

The following passage is from a recent book on the history of warfare.

19. The passage suggests that Henry V requests “not one man more” (line 17) because

(A) his strategy can work only with a small band of fighters

(B) he considers it more honorable to fight while outnumbered

(C) the opposing soldiers are unreliable

(D) no other fighters have the skills of the ones he has assembled

(E) he does not wish to be victorious

20. In line 26, the phrase “trotted out” most nearly means

(A) abused

(B) removed

(C) employed for rhetorical effect

(D) spared an indignity

(E) used flippantly

Damn the Torpedoes, Brian Burrell, McGraw-Hill, New York, 1999. Reproduced with permission of The McGraw-Hill Companies.

21. In line 34, the word “body” most nearly means

(A) stature

(B) strength

(C) corpse

(D) group

(E) anthology

22. In line 54, the word “charging” most nearly means

(A) accusing

(B) inspiring

(C) resting

(D) attacking

(E) prevailing

23. The passage indicates that the Battle of Agincourt and the Battle of Poitiers were similar in that in each case

I. the victorious army was the smaller

II. the French army was defeated

III. one side committed tactical errors

(A) I only

(B) I and II only

(C) I and III only

(D) I and III only

(E) I, II, and III

24. The passage suggests that the “whole story” (line 58) should include the possibility that

(A) numerical supremacy would not have been an advantage to the British

(B) King Henry had more soldiers available than was previously believed

(C) the English longbow was not as accurate as the French soldiers believed it to be

(D) confidence aided the French more than the British

(E) the French did not really outman the British

If you finish before time is called, you may check your work on this section only. Do not turn to any other section of the test.

SECTION 3
Time—25 minutes
20 questions


Turn to Section 3 of your answer sheet to answer the questions in this section.



Directions: For this section, solve each problem and decide which is the best of the choices given. Fill in the corresponding circle on the answer sheet. You may use any available space for scratchwork.



Notes

1. The use of a calculator is permitted.

2. All numbers used are real numbers.

3. Figures that accompany problems in this test are intended to provide information useful in solving the problems. They are drawn as accurately as possible EXCEPT when it is stated in a specific problem that the figure is not drawn to scale. All figures lie in a plane unless otherwise indicated.

4. Unless otherwise specified, the domain of any function f is assumed to be the set of all real numbers x for which f (x) is a real number.



Reference Information

The number of degrees of arc in a circle is 360.

The sum of the measures in degrees of the angles of a triangle is 180.


1. If n is 3 times an even number, then which of the following could be n?

(A) 14

(B) 15

(C) 16

(D) 17

(E) 18

2. A machine can produce 50 computer chips in 2 hours. At this rate, how many computer chips can the machine produce in 7 hours?

(A) 175

(B) 200

(C) 225

(D) 250

(E) 275

3. In the figure above, what is the value of x?

(A) 40

(B) 45

(C) 60

(D) 75

(E) 90

4. Any positive integer that is divisible by 6 and 15 must also be divisible by

(A) 12

(B) 21

(C) 30

(D) 72

(E) 90

5. If n percent of 20 is 4, what is n?

(A)

(B) 2

(C) 5

(D) 20

(E) 500

6. If , where n is a constant, and f (2) = 0, then

(A) –6

(B) –2

(C) 0

(D) 2

(E) 6

7. A square has the same area as a right triangle with sides of lengths 6, 8, and 10. What is the length of one side of the square?

(A) 4

(E) 12

8. If and , then which of the following is equivalent to ?

(A) 0

(B) 4 w

(C) –6 w

(D) 2 v

(E) –2v

9. If x is a negative number and , then which of the following must be true?

10. If , then

(A) 3

(B) 7

(C) 15

(D) 23

(E) 25

11. If then what is the smallest possible positive value of m?

(A) 6

(B) 6.5

(C) 7

(D) 7.5

(E) 8

12. Theo wants to buy a sweater that is priced at $60.00 before tax. The store charges a 6% sales tax on all purchases. If he gives the cashier $70.00 for the sweater, how much should he receive in change?

(A) $3.60

(B) $6.40

(C) $7.40

(D) $9.40

(E) $66.40

13. When m is subtracted from n, the result is r. Which of the following expresses the result when 2m is added to s?

14. In the figure above, the slope of line l is and the area of the triangle is 48 square units. What is the value of x + y?

(A) 13

(B) 14

(C) 19

(D) 22

(E) 96

15. Ellen takes a trip that is y miles long in total, where . She travels the first 15 miles at an average speed of 30 miles per hour and the rest of the trip at an average speed of 40 miles per hour. Which of the following represents the total time of the trip, in hours?

16. If y varies directly as m and inversely as the square of n, and if when and , then what is the value of y when and

(A) 0.125

(B) 0.25

(C) 0.5

(D) 1

(E) 2

17. If and , then which of the following expresses the value of ab in terms of s and t?

(A) st

18. If and y is greater than 1, then

(A) y1/12

(B) y1/7

(C) y7/12

(D) y7

(E) y12

19. In the figure above, if and , what is the area of the shaded region?

(A) 20

(B) 22

(C) 24

(D) 26

(E) 28

20. Every car at a certain dealership is either a convertible, a sedan, or both. If one-fifth of the convertibles are also sedans and one-third of the sedans are also convertibles, which of the following could be the total number of cars at the dealership?

(A) 28

(B) 29

(C) 30

(D) 31

(E) 32

If you finish before time is called, you may check your work on this section only. Do not turn to any other section of the test.

SECTION 4
Time—25 minutes
18 questions


Turn to Section 4 of your answer sheet to answer the questions in this section.



Directions: This section contains two types of questions. You have 25 minutes to complete both types. For questions 1–8, solve each problem and decide which is the best of the choices given. Fill in the corresponding circle on the answer sheet. You may use any available space for scratchwork.



Notes

1. The use of a calculator is permitted.

2. All numbers used are real numbers.

3. Figures that accompany problems in this test are intended to provide information useful in solving the problems. They are drawn as accurately as possible EXCEPT when it is stated in a specific problem that the figure is not drawn to scale. All figures lie in a plane unless otherwise indicated.

4. Unless otherwise specified, the domain of any function f is assumed to be the set of all real numbers x for which f (x) is a real number.



Reference Information

The number of degrees of arc in a circle is 360.

The sum of the measures in degrees of the angles of a triangle is 180.


1. A square has a perimeter of 36 centimeters. What is its area in square centimeters?

(A) 24

(B) 36

(C) 49

(D) 64

(E) 81

2. If b is a positive integer less than 100, then how many integer pairs (a, b) satisfy the equation

(A) 7

(B) 8

(C) 9

(D) 10

(E) 11

CLEANING COSTS IN THE McKENZIE OFFICE BUILDING

3. According to the table above, how much will it cost, in dollars, to clean each bathroom twice and each office once in the McKenzie Office Building?

(A) 200

(B) 400

(C) 450

(D) 600

(E) 850

4. If and , what is the value of a + b?

(A) 5

(B) 6

(C) 7

(D) 8

(E) 9

5. For all integers n greater than 1, let , where k is the sum of all the prime factors of n. What is the value of f (14) –f (6)?

(A) 4

(B) 5

(C) 6

(D) 9

(E) 14

6. The average (arithmetic mean) of four different positive integers is 20. What is the greatest possible value of any of these integers?

(A) 68

(B) 70

(C) 73

(D) 74

(E) 77

7. The radius of circle A is twice the radius of circle B. If the sum of their circumferences is 36π, then what is the radius of circle A?

(A) 9

(B) 12

(C) 14

(D) 16

(E) 18

8. The figure above shows a cube. How many different planes can be drawn such that each contains exactly two edges of the cube?

(A) 4

(B) 5

(C) 6

(D) 7

(E) 8

Directions: For student-produced response questions 9–18, use the grids at the bottom of the answer sheet page on which you have answered questions 1–8.

Each of the remaining ten questions requires you to solve the problem and enter your answer by marking the circles in the special grid, as shown in the examples below. You may use any available space for scratchwork.

• Mark no more than one circle in any column.

• Because the answer sheet will be machine-scored, you will receive credit only if the circles are filled in correctly.

• Although not required, it is suggested that you write your answer in the boxes at the top of the columns to help you fill in the circles accurately.

• Some problems may have more than one correct answer. In such cases, grid only one answer.

• No question has a negative answer.

Mixed numbers such as must be gridded as 3.5 or 7/2. (If is gridded, it will be interpreted as, not )

Decimal Answers: If you obtain a decimal answer with more digits than the grid can accommodate, it may be either rounded or truncated, but it must fill the entire grid. For example, if you obtain an answer such as 0.6666…, you should record your result as .666 or .667. A less accurate value such as .66 or .67 will be scored as incorrect.

Acceptable ways to grid are:

9. If 10 less than 2x is 22, then what is the value of x?

10. In the figure above, if , then what is the value of y?

11. If , then what is the value of

12. In the xy-plane, the line passes through the point (3, 5). What is the value of m?

13. The ratio of men to women in a room is 4:5. If the room contains three more women than men, how many women are in the room?

14. If, for some constant value b, the equation is satisfied by the point (5, 2), then what is one possible value of b?

15. A mixture of water and sucrose is 10% sucrose by weight. How many grams of pure sucrose must be added to a 200-gram sample of this mixture to produce a mixture that is 20% sucrose?

16. A runner runs a 16-mile race at an average speed of 8 miles per hour. By how many minutes can she improve her time in this race if she trains and increases her average speed by 25%?

17. The area of the figure above is 78. What is its perimeter?

18. Every sophomore at Hillside High School is required to study at least one language among Spanish, French, and Latin, but no one may study more than two. If 120 sophomores study Spanish, 80 study French, 75 study Latin, and 50 study two of the three languages, how many sophomores are there at Hillside High School?

If you finish before time is called, you may check your work on this section only. Do not turn to any other section of the test.

SECTION 5
Time—25 minutes
24 questions


Turn to Section 5 of your answer sheet to answer the questions in this section.



Directions: For each question in this section, select the best answer from among the choices given and fill in the corresponding circle on the answer sheet.



Each sentence below has one or two blanks, each blank indicating that something has been omitted. Beneath the sentence are five words or sets of words labeled A through E. Choose the word or set of words that, when inserted in the sentence, best fits the meaning of the sentence as a whole.

EXAMPLE:

Rather than accepting the theory unquestioningly, Deborah regarded it with -----.

(A) mirth

(B) sadness

(C) responsibility

(D) ignorance

(E) skepticism


1. The ------- with which the advisor managed the funds forced his clients to seek more reliable advice regarding investment.

(A) skill

(B) caution

(C) ineptitude

(D) recognition

(E) bitterness

2. As an Armenian born in Iran and educated in Lebanon, Vartan Gregorian brought ------- flavor to the presidency of Brown University that was unprecedented in the Ivy League.

(A) a perpetual

(B) an authoritative

(C) a structured

(D) an artificial

(E) a cosmopolitan

3. The lawyers did not have time to consider the contract in great detail; rather, they were able to give it only a ------- reading before they had to make their presentation on its merits.

(A) verbatim

(B) meandering

(C) tormented

(D) cursory

(E) substantial

4. The ------- in many parts of the city has made the ------- of infectious diseases more rapid, because pathogens spread quickly in close quarters.

(A) overcrowding . . propagation

(B) squalor . . circulation

(C) poverty . . deterioration

(D) congestion . . elimination

(E) proximity . . resilience

5. Much research in neuroscience today endeavors to ------- the mechanisms by which our brains turn the ------- data from our sense organs into coherent and understandable information.

(A) enhance . . quality of

(B) restore . . absence of

(C) enlighten . . source of

(D) attenuate . . dearth of

(E) elucidate . . deluge of


The passages below are followed by questions based on their content. Answer the questions on the basis of what is stated or implied in the passage and in any introductory material that may be provided.


Questions 6 and 7 are based on the following passage.

6. The “fruits” mentioned in line 10 represent

(A) spiritual growth

(B) artistic skill

(C) technological progress

(D) the means of acquiring food and shelter

(E) scientific knowledge

7. The “question” in line 16 is whether

(A) money can buy happiness

(B) intellectuals can earn a good living

(C) society can construct effective schools

(D) old ideas are relevant to modern society

(E) scholars are happier than merchants

Questions 8 and 9 are based on the following passage.

8. The “pointing of dogs” (line 7) is mentioned primarily as an example of

(A) an innate habit

(B) a behavior that humans find useful

(C) a skill that is hard to learn

(D) an ability that many other animals also have

(E) a skill that helps animals to find food

First passage: Jacob Feis. Shakespeare and Montaigne, c. 1890. Public domain

Second passage: Charles Darwin. The Expression of the Emotions in Man and Animals. 1872. Public domain

9. Which of the following best summarizes the main point of the paragraph?

(A) People will eat only what they are genetically determined to eat.

(B) All animal behavior is instinctive.

(C) Cows and other animals should not be fed by humans.

(D) Habits in animals are impossible to break.

(E) Inherited tendencies manifest themselves in behavioral habits.

Questions 10–16 are based on the following passage.

The following is an excerpt from an essay entitled Political Ideals, written in 1917 by Bertrand Russell.

10. Which of the following best summarizes the main point of the passage?

(A) People should strive harder to appreciate the arts.

(B) Nothing can be possessed exclusively by one person.

(C) Societies need strong laws against stealing.

(D) Creativity is of higher value than possessiveness.

(E) Scarce resources should be shared equally in a society.

11. The passage mentions “food and clothing” (lines 22–23) primarily as examples of things that

(A) everyone needs to survive

(B) create a positive atmosphere of sharing

(C) many underdeveloped countries lack

(D) cannot be shared as freely as other things

(E) are hard to find

12. As it is used in line 43, “such matters” can be inferred to refer to situations in which

(A) people must compete for ownership of goods

(B) artists struggle to sell their works

(C) people strive to be industrious

(D) philosophers endeavor to define human ideals

(E) possessing a good does not deny it to someone else

13. In line 51, the phrase “impulse of” most nearly means

(A) reaction against

(B) restriction of

(C) sharing of

(D) fear of

(E) desire for

14. According to the author, “force is impotent in such matters” (line 77) because

(A) violence cannot influence another person”s thoughts

(B) moral people do not engage in violence

(C) spiritual things cannot be acquired coercively

(D) a good person will always be protected by friends

(E) reason is more powerful than physical force

15. In the last paragraph, the author indicates that his thesis is not

(A) ancient

(B) a matter of logic

(C) relevant to those who are already happy

(D) original

(E) universal

16. Which of the following examples, if it existed, would most directly refute the main point of the author?

(A) a person who finds a large sum of money and gives it to charity

(B) an invention that benefits all of humankind even though it was created only to make money for its inventor

(C) a tyrant who murders intellectuals in order to maintain his authority

(D) a thief who steals in order to feed his starving family

(E) an army that invades another country and plunders its wealth

Questions 17–24 are based on the following passage.

The following passage was written for The Atlantic Monthly in 1902 by Native American writer Zitkala-Sa, also known as Gertrude Simmons Bonnin.

17. The main purpose of the passage as a whole is to

(A) describe one person”s perspective on an attempt at religious conversion

(B) compare Native American religious tradition to European religious tradition

(C) analyze the rise of Christianity in Native American tribes

(D) refute a misconception about the nature of Paganism

(E) describe a conflict between the author and her mother

18. The reference to “pitch and quality of voice” (lines 6–7) serves to emphasize

(A) the variety in vocal quality of religious singers

(B) the harshness with which many preachers rebuke their congregations

(C) the sounds that the author hears in nature

(D) the author”s inability to understand what the native preacher is saying

(E) the differences among members of the same race

19. In the first paragraph, the author characterizes the preacher primarily as

(A) respectful

(B) articulate

(C) uneducated

(D) intolerant

(E) compassionate

20. According to the passage, the preacher addressed the author as “cousin” because

(A) it is customary for preachers to refer to church members with that term

(B) the tribe members are all related

(C) the preacher”s mother and the author”s mother are sisters

(D) the preacher had forgotten the author”s name

(E) the author refused to answer to her given name

21. According to the passage, the native preacher and the author”s mother are alike in that they both

(A) have experienced attempted arson

(B) must travel a great deal

(C) have similar religious beliefs

(D) relish the midday meal

(E) enjoy excursions into the natural gardens

22. In line 68, the word “spirit” most nearly means

(A) apparition

(B) lively nature

(C) intent

(D) fear

(E) presence

23. In the final paragraph, the author characterizes herself primarily as

(A) mature

(B) creative

(C) vengeful

(D) repressed

(E) awed

24. The author mentions “conceptions of Infinite Love” (lines 71–72) in order to emphasize which of the following characteristics of the “pale-faced missionary” (lines 69–70)?

(A) small-mindedness

(B) reluctance to persist in the attempt to convert the author to Christianity

(C) generosity toward aborigines

(D) sympathy for animals

(E) high intelligence

If you finish before time is called, you may check your work on this section only. Do not turn to any other section of the test.

SECTION 6
Time—25 minutes
35 questions


Turn to Section 6 of your answer sheet to answer the questions in this section.



Directions: For each question in this section, select the best answer from among the choices given and fill in the corresponding circle on the answer sheet.



The following sentences test correctness and effectiveness of expression. Part of each sentence or the entire sentence is underlined; beneath each sentence are five ways of phrasing the underlined material. Choice A repeats the original phrasing; the other four choices are different. If you think the original phrasing produces a better sentence than any of the alternatives, select choice A; if not, select one of the other choices.

In making your selection, follow the requirements of standard written English; that is, pay attention to grammar, choice of words, sentence construction, and punctuation. Your selection should result in the most effective sentence—clear and precise, without awkwardness or ambiguity.

EXAMPLE:

The children couldn”t hardly believe their eyes.

(A) couldn”t hardly believe their eyes

(B) could hardly believe their eyes

(C) would not hardly believe their eyes

(D) couldn”t nearly believe their eyes

(E) couldn”t hardly believe his or her eyes


1. Claims about harmful effects of the genetic alteration of vegetables is more speculation than documented fact.

(A) is more speculation than documented fact

(B) are more with speculation than of a documented fact

(C) is more of a speculation than a documented fact

(D) are more speculation than documented fact

(E) are a matter of more speculation than documented fact

2.Having passed the test for certification, Mackenzie was looking forward to finding a challenging teaching position in her home town.

(A) Having passed

(B) Passing

(C) Being that she passed

(D) If she had passed

(E) For her passing

3. Having once been a provincial schoolmaster, Jean-Paul Sartre”s writing was always oriented more towards clear instruction than pontification.

(A) Jean-Paul Sartre”s writing was always oriented more towards clear instruction than pontification

(B) Jean-Paul Sartre always wrote more to instruct than to pontificate

(C) the writings of Jean-Paul Sartre were always oriented more toward instruction than pontification

(D) Jean-Paul Sartre was oriented in his writing more toward instruction than pontification

(E) Jean-Paul Sartre”s writing was more to instruct than to pontificate

4. Adam Smith was a professor of philosophy,a commissioner of customs, and founded the field of modern economics.

(A) a commissioner of customs, and founded the field of modern economics

(B) worked as commissioner of customs, and founded the field of modern economics

(C) a commissioner of customs, and the founder of the field of modern economics

(D) commissioned customs, and was the founder of the field of modern economics

(E) a commissioner of customs, and was the founder of the field of modern economics

5. John Locke was one of the first philosophers to attack the principle of primogeniture, the practice of handing the monarchy down to the king”s first-born son.

(A) primogeniture, the practice of handing the monarchy down

(B) primogeniture; the practice of handing the monarchy down

(C) primogeniture being the practice of handing the monarchy down

(D) primogeniture that which handed down the monarchy

(E) primogeniture this was the practice of handing the monarchy down

6. The nation”s fledgling economy struggled because the investment from other countries into its major industries was lacking from most of them.

(A) because the investment from other countries into its major industries was lacking from most of them

(B) because few other countries were willing to invest in its major industries

(C) due to the fact that few other countries would have invested in its major industries

(D) because of the lack of investment from few other countries in its major industries

(E) for the lack of investment in its major industries from other countries

7. The corporation began construction on the new building in January, but there is still no completion.

(A) there is still no completion

(B) they have yet to complete it

(C) it has yet to complete the project

(D) they have not still completed it yet

(E) it isn”t hardly done yet

8. Having spread more quickly than antibiotics could be distributed, doctors were prevented from effectively treating the virulent disease.

(A) doctors were prevented from effectively treating the virulent disease

(B) doctors could not effectively treat the virulent disease because it thwarted them

(C) the doctors who were trying to treat it effectively were prevented by the virulent disease

(D) the virulent disease prevented itself from its being treated effectively by the doctors

(E) the virulent disease prevented the doctors from treating it effectively

9. Although psychologist B. F. Skinner, who is best known as the man who popularized behaviorism, he also wrote a utopian novel entitled Walden Two.

(A) Skinner, who is best known as the man who popularized behaviorism, he

(B) Skinner, who is best known as the man who popularized behaviorism,

(C) Skinner is best known as the man who popularized behaviorism, he

(D) Skinner popularized behaviorism, for which he is well known, nevertheless he

(E) Skinner, who is best known as the man who popularized behaviorism, is the one who

10.Singing for over two hours, Anita”s hoarseness prevented her hitting the high notes.

(A) Singing for over two hours, Anita”s hoarseness prevented her hitting the high notes.

(B) Singing for over two hours, Anita was unable to hit the high notes because of her hoarseness.

(C) Having sung for over two hours, Anita”s hoarseness prevented her from hitting the high notes.

(D) Having sung for over two hours, Anita was no longer able to hit the high notes because of her hoarseness.

(E) Having sung for over two hours, Anita”s ability to hit the high notes was prevented by her hoarseness.

11. Some philosophers maintain that language is essential to formulating certain thoughts; others, that even the most complex thoughts are independent of words.

(A) thoughts; others, that

(B) thoughts, however, that others maintain that

(C) thoughts others suggest that

(D) thoughts and that others believe

(E) thoughts but others, however, that


The following sentences test your ability to recognize grammar and usage errors. Each sentence contains either a single error or no error at all. No sentence contains more than one error. The error, if there is one, is underlined and lettered. If the sentence contains an error, select the one underlined part that must be changed to make the sentence correct. If the sentence is correct, select choice E. In choosing answers, follow the requirements of standard written English.

EXAMPLE:

By the time the halfway point begun to hit their stride.


12. Ellen turned around and noticed that her was now

13. Marlena was honored not only for her in establishing the fund for war refugees but also so much and money

14. The Medieval era in music scholars during the reign of Pope Gregory and to have ended the 15th century.

15. Neither the artists at the vanguard of the Expressionist movement the critics could have foreseen of this new mode on the general public.

16. Several members that lowering the speed limit would not necessarily result in accidents.

17. By the time the operation five surgeons over 20 hours more than a dozen

18. Not until the recent scandal the newspapers published even vaguely about the company or

19. on a horse-drawn bus in Belgium in 1865, Friedrick Kekule had a to of the structure

20. The movement women”s issues as important a profound impact on the curricula today.

21. Legends and folk stories inevitably become transformed and as they are through the generations, often in order to changing political and

22. Although the remarks to the entire group, at the meeting could tell particularly intended

23. By all accounts, the restructuring of the federal department was unnecessary layers and dozens of

24. The professor that to attend the lecture next week 10 minutes

25. While in office a President pass more legislation, procedural obstacles, when the Congress and the administration are the the same political party.

26. A quick of Kurt”s art collection that a discerning eye for of art.

27. absent from the debate the vice president”s arrogance he typically displays forums.

28. Of the numerous strains of Streptococcus bacteria to cause infections, type B is the dangerous about birth.

29. Since 2001, the company employee training than in the previous 10 years


Directions: The following passage is an early draft of an essay. Some parts of the passage need to be rewritten.

Read the passage and select the best answers for the questions that follow. Some questions are about particular sentences or parts of sentences and ask you to improve sentence structure or word choice. Other questions ask you to consider organization and development. In choosing answers, follow the requirements of standard written English.


Questions 30–35 refer to the following passage.

(1) Most great scientists and artists are familiar with the so-called “eureka phenomenon.” (2) This is the experience that a thinker has when, after they thought about a problem long and hard, they suddenly come upon a solution in a flash when they are no longer thinking about it. (3) The name of the phenomenon comes from the legend of Archimedes. (4) He had been thinking for days about a hard problem that had come from the king, King Hieron II. (5) The problem was how to determine whether the king”s crown was pure gold without destroying it. (6) As he was bathing, the solution to the problem came to Archimedes in a flash and he ran naked through the streets of Syracuse shouting “Eureka!” meaning “I have found it!”

(7) Students should understand this also. (8) You have probably had the experience of thinking about a paper or a math problem for so long that it”s like one”s brain gets frozen. (9) When this happens, it is best to get away from the problem for a while rather than obsess about it. (10) Isaac Asimov, one of the most prolific writers of all time, used to go to the movies every time he got writer”s block. (11) He claimed that he always came out of the movie knowing exactly how to get his story back on track.

(12) Unfortunately, many students today don”t have time for that. (13) They feel so much pressure to get everything done—their homework, their jobs, their sports, their extracurricular activities—that they think that taking “time out” to relax their brains is just a costly waste of time. (14)This is really too bad because very often relaxation is more valuable to a student than just more hard work.

30. Which of the following is the best revision of the underlined portion of sentence 2 (reproduced below)?

This is the experience that a thinker has when, after they thought about a problem long and hard, they suddenly come upon a solution in a flash when they are no longer thinking about it.

(A) that a thinker has when, after they thought long and hard about a problem, their solution suddenly arises like a flash

(B) that thinkers have when a solution suddenly had arisen like a flash after they were thinking long and hard about a problem

(C) that a thinker has when, after having thought long and hard about a problem, they suddenly come upon a solution

(D) that thinkers have when, after having thought long and hard about a problem, they suddenly come upon a solution

(E) that thinkers have when, thinking long and hard about a problem, they suddenly come upon a solution in a flash

31. Which of the following is the best way to combine sentences 3, 4, and 5?

(A) The name of the phenomenon comes from the legend of Archimedes, who had been thinking for days about how to determine whether King Hieron II”s crown was pure gold without destroying it.

(B) Archimedes had been thinking for days about how to determine whether King Hieron II”s crown was pure gold without destroying it, and this is where the name of the phenomenon comes from.

(C) The legend of Archimedes thinking about how to determine whether King Hieron II”s crown was pure gold without destroying it is the origin of the name of the phenomenon.

(D) The phenomenon is named for Archimedes and his thinking for days about how to determine whether King Hieron II”s crown was pure gold without destroying it.

(E) The name of the phenomenon was from Archimedes, and his thinking for days about how to determine without destroying it whether King Hieron II”s crown was pure gold.

32. Which of the following revisions of sentence 7 most clearly and logically introduces the second paragraph?

(A) This historical episode is something that all students should learn about in school.

(B) Understanding this phenomenon may help students to improve their studies.

(C) Nevertheless, this episode is something that all students should know.

(D) Understanding this episode requires a more thorough understanding of its historical setting.

(E) Many have tried to understand this phenomenon, but few have succeeded.

33. Which of the following is the best revision of the underlined portion of sentence 8 (reproduced below)?

You have probably had the experience of thinking about a paper or a math problem for so long that it”s like one”s brain gets frozen.

(A) it seems that your brain gets frozen

(B) one”s brain gets frozen

(C) your brain seems to freeze

(D) your brains seem to freeze

(E) one”s brain seems to freeze

34. Where is the best place to insert the following sentence?

Perhaps if students could work such little excursions into their busy study schedules, they would have similar “eureka” experiences.

(A) after sentence 7

(B) after sentence 8

(C) after sentence 9

(D) after sentence 10

(E) after sentence 11 (as the last sentence of the second paragraph)

35. In context, which of the following revisions of the underlined portion of sentence 12 (reproduced below) is most effective at making it clearer and more specific?

Unfortunately, many students today don”t have time for that.

(A) today have hardly even 1 hour for such things

(B) today, unlike those in Archimedes” time, don”t have time to go to the movies

(C) today don”t have time for such excursions

(D) of modern times lack sufficient time for the kinds of things explained above

(E) today lack sufficient time for things like this

If you finish before time is called, you may check your work on this section only. Do not turn to any other section of the test.

SECTION 7
Time—20 minutes
16 questions


Turn to Section 7 of your answer sheet to answer the questions in this section.



Directions: For this section, solve each problem and decide which is the best of the choices given. Fill in the corresponding circle on the answer sheet. You may use any available space for scratchwork.



Notes

1. The use of a calculator is permitted.

2. All numbers used are real numbers.

3. Figures that accompany problems in this test are intended to provide information useful in solving the problems. They are drawn as accurately as possible EXCEPT when it is stated in a specific problem that the figure is not drawn to scale. All figures lie in a plane unless otherwise indicated.

4. Unless otherwise specified, the domain of any function f is assumed to be the set of all real numbers x for which f (x) is a real number.



Reference Information

The number of degrees of arc in a circle is 360.

The sum of the measures in degrees of the angles of a triangle is 180.


1. If four apples cost 20 cents, then, at this rate, how much would ten apples cost?

(A) $.40

(B) $.50

(C) $.60

(D) $.70

(E) $.80

2. If 2b = 8, then 3b =

(A) 6

(B) 9

(C) 27

(D) 64

(E) 81

3. How much greater is the average (arithmetic mean) of a, b, and 18 than the average of a, b, and 12?

(A) 2

(B) 3

(C) 4

(D) 5

(E) 6

4. The first day of a particular month is a Tuesday. What day of the week will it be on the 31st day of the month?

(A) Wednesday

(B) Thursday

(C) Friday

(D) Saturday

(E) Sunday

5. How many integer pairs (m, n) satisfy the statements and

(A) 5

(B) 6

(C) 7

(D) 8

(E) more than 8

6. If y% of 50 is 32, then what is 200% of y?

(A) 16

(B) 32

(C) 64

(D) 128

(E) 256

7. For x > 0, the function g (x) is defined by the equation g (x) = x + x1/2. What is the value of g (16)?

(A) 16

(B) 20

(C) 24

(D) 64

(E) 272

8. In the figure above, if the slope of is –¾, what is the area of ΔABO?

(A) 54

(B) 72

(C) 96

(D) 108

(E) 192

–1, 1, 2,–1, 1, 2,–1, 1, 2, …

9. The sequence above continues according to the pattern shown. What is the sum of the first 25 terms of this sequence?

(A) 15

(B) 16

(C) 18

(D) 19

(E) 21

10. A jar contains only white and blue marbles of identical size and weight. The ratio of the number of white marbles to the number of blue marbles is 4 to b. If the probability of choosing a white marble from the jar at random is ¼, then what is the value of b?

(A) 1

(B) 2

(C) 6

(D) 12

(E) 16

11. The area of a right triangle is 10 square centimeters. If the length of each leg, in centimeters, is a positive integer, then what is the least possible length, in centimeters, of the hypotenuse?

12. If y is a number less than 0 but greater than –1, which of the following expressions has the greatest value?

(A) 100 y

(B) y2

(C) y3

(D) y4

(E) y5

13. If at least one wuzzle is grumpy, then some fuzzles are lumpy.

If the statement above is true, then which of the following must also be true?

(A) If all wuzzles are grumpy, then all fuzzles are lumpy.

(B) If no wuzzle is grumpy, then all fuzzles are lumpy.

(C) If all fuzzles are lumpy, then all wuzzles are grumpy.

(D) If no wuzzle is grumpy, then no fuzzle is lumpy.

(E) If no fuzzle is lumpy, then no wuzzle is grumpy.

14. Six buses are to carry 200 students on a field trip. If each bus must have no more than 40 students and no fewer than 30 students, then what is the greatest number of buses that can have 40 students?

(A) 6

(B) 5

(C) 4

(D) 3

(E) 2

15. The volume of right cylinder A is twice the volume of right cylinder B. If the height of cylinder B is twice the height of cylinder A, then what is the ratio of the radius of cylinder A to the radius of cylinder B?

(A) 1 to 2

(B) 1 to 1

(D) 2 to 1

(E) 4 to 1

16. In a garden that is divided into x rows of x squares each, w of the squares lie along the boundary of the garden. Which of the following is a possible value for w?

(A) 29

(B) 34

(C) 40

(D) 46

(E) 55

If you finish before time is called, you may check your work on this section only. Do not turn to any other section of the test.

SECTION 8
Time—20 minutes
19 questions


Turn to Section 8 of your answer sheet to answer the questions in this section.



Directions: For each question in this section, select the best answer from among the choices given and fill in the corresponding circle on the answer sheet.



Each sentence below has one or two blanks, each blank indicating that something has been omitted. Beneath the sentence are five words or sets of words labeled A through E. Choose the word or set of words that, when inserted in the sentence, best fits the meaning of the sentence as a whole.

EXAMPLE:

Rather than accepting the theory unquestioningly, Deborah regarded it with -----.

(A) mirth

(B) sadness

(C) responsibility

(D) ignorance

(E) skepticism


1. The evidence for ESP is ------- at best, so very few reputable scientists are willing to even ------- that the phenomenon exists.

(A) meager . . regret

(B) unconvincing . . suggest

(C) plentiful . . admit

(D) paltry . . deny

(E) strong . . assume

2. The concept that the Earth is round was once ------- theory, but is now accepted as an inarguable truth.

(A) an incontrovertible

(B) a mellifluous

(C) an admirable

(D) a dubious

(E) an accurate

3. The controversy within the party produced a ------- that broke it into several factions even before the matter could be fully discussed among the members.

(A) unanimity

(B) schism

(C) caucus

(D) commemoration

(E) prognostication

4. Horace Mann, widely acknowledged as the father of American public schooling, ------- the Massachusetts legislature to institute a system for ------- universal access to education.

(A) petitioned . . restricting

(B) established . . denying

(C) persuaded . . ensuring

(D) tolerated . . requiring

(E) discouraged . . vouchsafing

5. The light from most stars takes millions of years to reach us, so not only is the present existence of these stars -------, but so are the very concepts of “the present” and “existence.”

(A) debatable

(B) methodical

(C) indecorous

(D) imperious

(E) profuse

6. Although many parents prefer to be ------- when their children broach sensitive personal subjects, others resort instead to ------- so as to make any potentially offensive matters seem less objectionable.

(A) honest . . anachronism

(B) intolerant . . laudation

(C) clandestine . . obligation

(D) candid . . euphemism

(E) forthright . . coercion


The passages below are followed by questions based on their content; questions following a pair of related passages may also be based on the relationship between the paired passages. Answer the questions on the basis of what is stated or implied in the passage and in any introductory material that may be provided.


Questions 7–19 are based on the following passages.

The following two passages concern the use of “reinforcers,” which are rewards or punishments used to encourage desired behaviors, and “contingencies,” which are the arrangements of those reinforcers to shape behavior.

First passage: B. F. Skinner, The Technology of Teaching, © 1968 Prentice-Hall.

Second passage: © 2004 Christopher Black. All rights reserved. Reprinted by permission of the author.

7. It can be inferred that the “English gentleman” (line 5) believed that good teaching utilized

(A) punishment

(B) well-written books

(C) reward

(D) humor

(E) careful grading

8. The parenthetical remark in lines 17–19 is intended to caution educators against

(A) failing to make grades and diplomas meaningful to students

(B) punishing students unnecessarily

(C) employing dull lessons

(D) emphasizing entertainment over rigor

(E) using rewards as reinforcers

9. Passage 1 indicates that “cultures and epochs” (lines 22–23) vary in the ways that

(A) universities choose from among their applicants

(B) academic awards are effective as motivators

(C) universities teach literature

(D) students are paid money for learning

(E) the media portray educational crises

10. The Wilde story in lines 23–32, “In 1876 … everyone I knew,”” is intended to illustrate

(A) how the modern cultural perception of academic honors differs from that of a previous era

(B) a particularly effective teaching strategy

(C) how a famous author used rewards to teach his students

(D) the dangerous effects of using academic rewards

(E) the point that Plato makes in the first sentence

11. Passage 1 mentions which of the following as “problems” (line 37) inherent in the use of positive reinforcers in education?

I. difficulties in scheduling the reinforcers

II. limitations in the supply of reinforcers

III. the fact that rewards encourage only superficial learning

(A) I only

(B) II only

(C) I and II only

(D) I and III only

(E) I, II, and III

12. In the final paragraph of Passage 1, the author suggests that grades are problematic as reinforcers because they

(A) cannot be given to every student

(B) do not provide sensual gratification, as food does

(C) are not publicized enough

(D) are not given immediately after the desired behavior is exhibited

(E) are not as useful to the student as money

13. The sentence that begins on line 78, “Reasoning sometimes works … on deaf ears,” is intended to describe the interaction between

(A) those who promote the use of punishments and those who oppose it

(B) educators and philosophers

(C) parents and teachers

(D) teachers and administrators

(E) teachers and students

14. In Passage 2, Alfie Kohn and Edward Deci (lines 90–91) are mentioned as examples of

(A) teachers who use rewards as reinforcers

(B) experts who question the effectiveness of rewards as reinforcers

(C) scientists on opposite sides of a debate

(D) educators who prefer negative reinforcers to positive reinforcers

(E) educators who advocate a careful schedule of contingencies for students

15. In saying that “the introduction of a reward system … changes the nature of the desired behavior” (lines 92–95), the author of Passage 2 indicates that

(A) many people object to the use of punishments in school

(B) teachers find it difficult to find the right kinds of rewards for student performance

(C) experts disagree about the effects of rewards on human behavior

(D) such systems tend to decrease student interest in the activity for its own sake

(E) not enough study has been done on the effectiveness of rewards in education

16. Deci”s conclusion about the experiment described in Passage 2 (lines 96–110) assumes that the subjects in the study

(A) are well educated

(B) are highly proficient at solving puzzles

(C) have not participated in reward systems before

(D) can make inferences about the motives of the experimenter

(E) have some teaching experience

17. The author of Passage 2 mentions that “the human mind is an enormously complex machine” (lines 129–130) in order to suggest that

(A) a simplistic theory about the effectiveness of rewards is unwise

(B) people cannot be easily fooled

(C) many learning disabilities require special attention

(D) teachers often find it hard to teach certain subjects

(E) Deci”s experiment was poorly constructed

18. The description of the “problems” (line 37) with positive reinforcers in Passage 1 would most likely be regarded by Edward Deci as

(A) thorough and fair

(B) presumptuous and incomplete

(C) unfair to educators

(D) erroneous in concluding that the methods of the “gentleman” were ineffective

(E) likely correct, but worthy of further study

19. Which of the following assumptions is shared by the authors of both passages?

(A) Rewards are ineffective as reinforcers of behavior.

(B) Honors and grades are necessary elements of institutional education.

(C) Good teaching is always focused on play.

(D) Negative feedback is not an effective teaching tool.

(E) If prizes are to be used in a classroom, there must be enough for all students.

If you finish before time is called, you may check your work on this section only. Do not turn to any other section of the test.

SECTION 9
Time—10 minutes
14 questions


Turn to Section 9 of your answer sheet to answer the questions in this section.



Directions: For each question in this section, select the best answer from among the choices given and fill in the corresponding circle on the answer sheet.



The following sentences test correctness and effectiveness of expression. Part of each sentence or the entire sentence is underlined; beneath each sentence are five ways of phrasing the underlined material. Choice A repeats the original phrasing; the other four choices are different. If you think the original phrasing produces a better sentence than any of the alternatives, select choice A; if not, select one of the other choices.

In making your selection, follow the requirements of standard written English; that is, pay attention to grammar, choice of words, sentence construction, and punctuation. Your selection should result in the most effective sentence—clear and precise, without awkwardness or ambiguity.

EXAMPLE:

The children couldn”t hardly believe their eyes.

(A) couldn”t hardly believe their eyes

(B) could hardly believe their eyes

(C) would not hardly believe their eyes

(D) couldn”t nearly believe their eyes

(E) couldn”t hardly believe his or her eyes


1.Choreographer Alvin Ailey”s works, whose style is rooted in the techniques of modern dance, jazz dance and ballet, draw upon African American themes.

(A) Choreographer Alvin Ailey”s works, whose style is rooted in the techniques of modern dance, jazz dance and ballet, draw upon African American themes.

(B) Alvin Ailey has a style of a choreographer that is rooted in the techniques of modern dance, jazz dance and ballet of which also draws upon African American themes.

(C) The works of choreographer Alvin Ailey, which draw upon African American themes, have a style that is rooted in the techniques of modern dance, jazz dance, and ballet.

(D) Choreographer Alvin Ailey”s works, which have a style that is rooted in the techniques of modern dance, jazz dance, and ballet, drawing upon African American themes.

(E) Alvin Ailey”s style, a choreographer, is rooted in the techniques of modern dance and jazz dance and ballet which also draws upon African American themes.

2. The mountain climbers getting this far, they did not want to return without having reached the peak.

(A) The mountain climbers getting this far, they

(B) Having gotten this far, the mountain climbers

(C) To have gotten this far, the mountain climbers

(D) The mountain climbers having gotten so far that they

(E) Mountain climbers getting this far

3. Although usually even-tempered,Rachel”s irritation with her supervisor caused her to become uncharacteristically cantankerous.

(A) Rachel”s irritation with her supervisor caused her to become

(B) Rachel being irritated by her supervisor caused her to become

(C) Rachel was irritated by her supervisor, and so became

(D) her supervisor caused Rachel through irritation to become

(E) Rachel, due to her supervisor”s irritation, caused her to become

4. Alberta had a high opinion of her own abilities, and so believed that her salary should be higher than the other agents in her firm.

(A) should be higher than

(B) should be higher than those of

(C) being higher than

(D) would have been highest of

(E) should have to be highest of

5. The police chief was hoping that by assigning an extra officer to the patrol he would decrease the amount of elicit behavior in the neighborhood.

(A) would decrease the amount of elicit

(B) would be able to decrease the elicit

(C) would decrease the amount of illicit

(D) might be able to lessen that of the illicit

(E) decreases the amount of illicit

6. Watching from the balcony, the paraders marched triumphantly through the streets below us.

(A) Watching from the balcony

(B) While watching from the balcony

(C) As we had been watching from the balcony

(D) As we watched from the balcony

(E) From the balcony, while watching

7. By the time we arrived at the campsite where the troop would be staying,the counselors set up all the tents.

(A) the counselors set up all the tents

(B) setting up all the tents were the counselors

(C) set up by the counselors are the tents

(D) the tents are set up by the counselors

(E) the counselors had set up all the tents

8. For the foreseeable future, neither Matt nor David are permitted to use the pool without first getting permission from a parent.

(A) neither Matt nor David are permitted to use the pool without

(B) Matt nor David can be permitted to use the pool without

(C) neither Matt nor David is permitted to use the pool without

(D) neither Matt or David is permitted for using the pool without

(E) neither Matt nor David is permitted to use the pool by

9. An outstanding tennis player, Erica was concerned not only with working her way to the top of the national rankings, but also wanted to compete with class and dignity.

(A) also wanted to compete with class and dignity

(B) also with competing with class and dignity

(C) also with wanting to have competed with class and dignity

(D) she also wanted to compete with class and dignity

(E) she was also wanting to compete with class and dignity

10. Roberto volunteered to be an usher, not wanting to be the one that had to clean up the petals after the ceremony.

(A) that had to clean up the petals after the ceremony

(B) which had to clean up the petals after the ceremony

(C) who had to clean up the petals after the ceremony

(D) that was cleaning the petals up after the ceremony

(E) who was to be cleaning the petals after the ceremony

11. Rebecca liked to read books, of which she found autobiographies being the most interesting.

(A) books, of which she found autobiographies being the most interesting

(B) books, the most fascinating of which to her she found the autobiographies

(C) books, autobiographies being the most interesting she found

(D) books; she found autobiographies to be the most interesting

(E) books, to which autobiographies were the most interesting

12. Forced to live apart from his family and to move from place to place to avoid detection by the government”s ubiquitous informers, St. Pierre adopting a number of disguises.

(A) St. Pierre adopting a number of disguises

(B) St. Pierre having adopted a number of disguises

(C) had adopted for St. Pierre a number of disguises

(D) a number of disguises by St. Pierre had adopted

(E) St. Pierre had to adopt a number of disguises

13. The Santa Catalina Mountains, forming twelve million years ago during a period when the Western North American Continent was stretching, cracking into blocks bordered by steep faults.

(A) Mountains, forming twelve million years ago during a period when the Western North American Continent was stretching

(B) Mountains were formed twelve million years ago during a period when the Western North American Continent was being stretched

(C) Mountains, having been formed twelve million years ago during a period when the Western North American Continent was stretching

(D) Mountains was formed twelve million years ago during a period when the Western North American Continent was being stretched

(E) Mountains had been formed during a period twelve million years ago when the Western North American Continent was stretching

14. The most challenging aspect of the project is we have to coordinate our work carefully.

(A) we have to coordinate our work carefully

(B) we must coordinate our work carefully

(C) our coordination of our work carefully

(D) coordinating our work carefully

(E) in careful coordination of our work

If you finish before time is called, you may check your work on this section only. Do not turn to any other section of the test.

ANSWER KEY

SCORE CONVERSION TABLE

How to score your test

Use the answer key on the previous page to determine your raw score on each section. Your raw score on each section except Section 4 is simply the number of correct answers minus ¼ of the number of wrong answers. On Section 4, your raw score is the sum of the number of correct answers for questions 1–18 minus ¼ of the number of wrong answers for questions 1–8. Next, add the raw scores from Sections 2, 5, and 8 to get your Critical Reading raw score, add the raw scores from Sections 3, 4, and 7 to get your Math raw score, and add the raw scores from Sections 6 and 9 to get your Writing raw score. Write the three raw scores here:

Raw Critical Reading score: ____________ Raw Math score: ____________ Raw Writing score: ___________

Use the table below to convert these to scaled scores.

Scaled scores: Critical Reading: _____________ Math: _____________ Writing: _____________

SCORE CONVERSION TABLE FOR WRITING COMPOSITE
[ESSAY + MULTIPLE CHOICE]

Calculate your Writing raw score as you did on the previous page and grade your essay from a 1 to a 6 according to the standards that follow in the detailed answer key.

Essay score: ____________ Raw Writing score: ____________

Use the table below to convert these to scaled scores.

Scaled score: Writing: _____________

Detailed Answer Key

Section 1


Consider carefully the issue discussed in the following passage, then write an essay that answers the question posed in the assignment.


The best leaders are not those who seek power or have great political skill. Great leaders—and these are exceptionally rare, especially today—represent the best selves of the people they represent.


Assignment: What are the most important qualities of a leader? Write an essay in which you answer this question and discuss your point of view on this issue. Support your position logically with examples from literature, the arts, history, politics, science and technology, current events, or your experience or observation.



The following essay received 6 points out of a possible 6. This means that, according to the graders, it

• develops an insightful point of view on the topic

• demonstrates exemplary critical thinking

• uses effective examples, reasons, and other evidence to support its thesis

• is consistently focused, coherent, and well organized

• demonstrates skillful and effective use of language and sentence structure

• is largely (but not necessarily completely) free of grammatical and usage errors


There is no more important decision that a citizen can make than one”s choice of a leader. I am inclined to agree with Thomas Hobbes, who believed that humans are hardly better than other mammals without a social contract that binds us to work together as a society. Artists could not survive in a society that does not provide a means of trading art for food. Great teachers cannot survive in a society without a means of trading wisdom for shelter. This requires a social order, a division of labor, and a group we call leaders. Yet we know that power corrupts, and absolute power corrupts absolutely. So how do we maintain a just society when we must bestow corrupting powers upon members of that society?

Those who seek power are too often not our best leaders, but rather our best politicians. George Bush, John F. Kennedy and Ronald Reagan came to power not so much because of their visionary leadership but because of their appeal to a television-viewing audience. The problems with democracy are well known. In order to become elected, most politicians must appeal to a broad range of citizens. To gain this appeal, they must pander to their constituents, and often take conflicting or equivocal stances on issues. Of course, the politicians claim that they are taking “forceful stances” to “bring the people together.” But it is far more likely that they are simply doing their best to make everyone happy without putting their feet in their mouths.

So why is democracy the best way of electing a leader? Because the alternatives are much worse. To gain power, one must either use force or pander to those who do. Which is a better alternative? A country is weak if its people do not support it, and, at the very least, a democracy can claim a good degree of public support. Even more importantly, only a democracy allows for the possibility of finding a reluctant leader with genuine leadership skills. It doesn”t happen often enough, but when it does, it is breathtaking. Witness the phenomenon of Howard Dean”s campaign for the 2004 Democratic nomination for president, or Ross Perot”s run in 1992. Neither was ultimately successful, but both demonstrated the potential of motivated citizens to change their country.

Without democracy, there is no hope for an ordinary citizen to change his or her country. What makes America great is not that its policies are always correct. Indeed, they are often deeply flawed. What makes America great is that it is run by those who are not even seeking power: the citizens.


The following essay received 4 points out of a possible 6, meaning that it demonstrates adequate competence in that it

• develops a point of view on the topic

• demonstrates some critical thinking, but perhaps not consistently

• uses some examples, reasons, and other evidence to support its thesis, but perhaps not adequately

• shows a general organization and focus, but shows occasional lapses in this regard

• demonstrates adequate but occasionally inconsistent facility with language

• contains occasional errors in grammar, usage, and mechanics


Someone once said that great men don”t seek greatness but have it thrust upon them. I think this is true, because those who have really changed the world were not slick politicians but rather people who had such great leadership skill and charisma that others forced them into leadership roles. Good examples of this are Jesus, Mahatma Gandhi, Mother Theresa and George Washington.

After his great victories in the American Revolutionary War against Great Britain, George Washington wanted to retire to his farm in Virginia and live out the rest of his days as a humble farmer. He did not want to become the political leader of a brand new country. But the Continental Congress looked to him for leadership, and sought him out to be the first

President of the United States. Washington saw that his country needed him and answered the call.

Similarly, Mahatma Gandhi did not seek personal power, but only justice for his people. His humility and selflessness are what made him one of the great leaders of the twentieth century, and a model for the cause of nonviolent activism.

It is unfortunate that today only millionaires with big political connections seem to have any chance at being elected to national office. Maybe they have a shot at a local race, but the congress and the presidency seem to be off limits. The answer is to get more involved in politics yourself, as a voter, and avoid voting for candidates just because they are popular but instead because they have good souls.


The following essay received 2 points out of a possible 6, meaning that it demonstrates some incompetence in that it

• has a seriously limited point of view

• demonstrates weak critical thinking

• uses inappropriate or insufficient examples, reasons, and other evidence to support its thesis

• is poorly focused and organized and has serious problems with coherence

• demonstrates frequent problems with language and sentence structure

• contains errors in grammar and usage that seriously obscure the author”s meaning


I”m not sure how it can be that you can be the best person to be in power if you don”t want to be. In this country, at least, running for president or something like that takes a lot of effort, and I think you have to be a really hard worker in order to become president or senator.

An example of somebody who is a hard worker who got into office is former president Bill Clinton. Although many people think he had indiscretions in office, he came from a very poor family where he was only raised by his mother because his father left the family when he was young. He worked really hard and became a Rhodes scholar and was elected as governor at a very young age. He knew even when he was a very young kid that he wanted to become a great leader like John F. Kennedy.

Clinton was a good leader because he understood where a lot of people were coming from. He wasn”t just a rich guy who got into office because he had rich relatives who got him there. I don”t think you can say that the best leaders are the ones who don”t want to be in office. If you didn”t want to be in office, then you shouldn”t run.

Section 2

1. A Alisha was holding a grudge, which is a feeling of resentment.

resentment = ill will; fortitude = strength of mind to endure; sarcasm = wit used to ridicule; elation = extreme joy

2. C There were people who expected the governor to be inarticulate (unable to speak clearly), so they would be surprised if he were articulate. intolerance = inability to put up with something; fatigue = tiredness; eloquence = persuasiveness in speech; endurance = ability to last, often through hard times

3. D The language of commoners would be logically described as common. But the novelists preferred another kind of parlance (speech): that of the upper classes. A word such as elegant would work nicely. elite = superior; sympathetic = compassionate; colloquial = characteristic of everyday language; refined = precise, elegant; utilitarian = practical, stressing utility

4. A The second half of this sentence presents a definition. The word in the blank should mean “exploring the world.” peripatetic = walking from place to place; conventional = customary; tolerant = willing to put up with something; coordinated = well-matched; remunerative = profitable

5. E A position that requires public speaking would be difficult for a person who does not like to speak or is afraid of crowds. vivacious = full of life; garrulous = talkative; amiable = friendly; reticent = hesitant to share one”s feelings or opinions with others

6. C The tickbird gets something from the hippopotamus, and the hippopotamus gets something from the tickbird; it”s a give-and-receive relationship. deteriorating = diminishing in quality; symbiotic = of mutual benefit; regressive = going backwards; vacillating = going back and forth

7. A This sentence establishes a contrast between how modern scientists think and how early philosophers thought. The contrast shows that the early philosophers were not using experiments as much as their own minds to draw conclusions and that the modern scientists rely more on experimental data to draw their conclusions. empirical = relying on the observations made from experiments; coercion = pressure on someone to act; deduction = reaching a conclusion through the use of logic; clerical = relating to office work; intuitive = known innately

8. B The first blank should be a word like merging or unification, because many companies are under a single owner. This would be troublesome to those who value independence. retraction = taking something back; differentiation = finding a difference between two things; consolidation = combining of multiple things into one common entity; collaboration = working together on something; dissemination = the spread of something

9. E Passage 2 distinguishes between education and schooling. It states that the main product of schooling is not education (lines 15–16) and that the struggle that defines education is denied by schooling (lines 22–23). Passage 1 makes no such distinction, and speaks of education as if it is inseparable from the idea of schooling.

10. E The passage mentions that education would diminish social distinctions (“obliterate factitious distinctions in society” (lines 13–14)), improve living standards (“prevents being poor” (line 8)), provide the means to counteract greed (“resist the selfishness of other men” (lines 5–6)), and increase self-sufficiency (“gives each man the independence” (line 4)). It does not, however, mention anything about reducing crime.

11. A The passage suggests that education is the great equalizer and that the spread of education will open a wider area over which the social feelings will expand. It concludes by commenting that if this education should be universal and complete it would obliterate factitious distinctions in society.

12. B Passage 2 states that education, which is the acquisition of competence, power, wisdom and discernment (lines 19–20), is achieved only through the struggle for sense in the world (lines 21–22). Therefore, this struggle is empowering.

13. A “The Beginnings of the Scientific Method” is the best title, because this passage begins by discussing the scientists of the Renaissance and how they brought about the most fundamental alterations in the world of thought … by devising a new method for discovering knowledge (lines 1–5). This new method was the scientific method.

14. C Saying that the early modern scientists laid greatest stress upon observation and the formation of temporary hypotheses (lines 7–9) is like saying they emphasized observation and hypotheses.

15. C In lines 19–21 the passage suggests that earlier scientists were simply trying to find the confirmation of Biblical statements about the firmament.

16. D Choice II is confirmed in lines 32–34: The principle of the barometer was discovered by Galileo”s pupil(student) Torricelli. Choice III is confirmed in lines 41–42: Galileo discovered the moons around Jupiter.

17. E The final paragraph states that Renaissance scientists believed that everything consists of bodies in motion, that everything conforms to a mechanical model. The heavens above and the smallest particles below all exhibit the same laws of motion—even, as it says in the next sentence,human thought (lines 67–71).

18. C The final paragraph discusses how the scientific method changed the way science was done.

19. B The passage mentions in lines 22–24 that many military leaders cement their solidarity by reveling (taking delight) in their numerical disadvantage. They considered it more honorable to fight with fewer men and beat a larger opponent.

20. C Stating that a well-known proverb was trotted out in many instances of the glorious, fighting few (lines 25–27), in this context, is like saying that the proverb was used for rhetorical effect because it was used to persuade and inspire the troops.

21. D When the prince says that we be a small body when compared to the army of our enemies, he is saying that they are a small army or group of men.

22. D This sentence is discussing the tactical errors of the French in two different battles. The phrase charging before they were ready simply means attacking before they were ready.

23. E All three of these facts are true and are mentioned in the passage.

24. A The passage states in the final paragraph that ten thousand more men might actually have hindered the English (lines 59–60) and that it seems that in fact … strength is not always proportional to size (lines 62–64).

Section 3

1. E Since n is equal to 3 times an even number, you can eliminate any answer choice that is not a multiple of 3 (A, C, and D). Answer choice (B): 15 = 3 × 5; 5 is an odd number, so this answer choice is out. Answer choice (E): 18 = 3 × 6; 6 is an even number.

(Chapter 9, Lesson 3: Numerical Reasoning Problems)

(Chapter 7, Lesson 4: Ratios and Proportions)

3. C Angles that form a straight angle have a sum of 180°:

(Chapter 10, Lesson 1: Lines and Angles)

4. C Find the smallest number that is divisible by both 15 and 6 and see which answer choice works.

Multiples of 15: 15, 30, 45, …

Multiples of 6: 6, 12, 18, 24, 30, …

(Chapter 7, Lesson 7: Divisibility)

5. D n % of 20 is 4

(Chapter 7, Lesson 5: Percents)

(Chapter 11, Lesson 2: Functions)

7. D First find the area of the right triangle:

Area = ½(base)(height)

Area = ½(8)(6) = 24

Next, set up an equation for the area of a square.

(Chapter 10, Lesson 5: Areas and Perimeters)

Alternatively, you can try finding values for v and w that work, like 1 and 4, and plug them into and into the choices and find the match. (Chapter 8, Lesson 1: Solving Equations)

Interpret the absolute value: OR You are told that x is negative, so is the answer. (Chapter 8, Lesson 6: Inequalities, Absolute Values, and Plugging In)

When evaluating –x2, don”t forget to square the value before taking its opposite!

(Chapter 8, Lesson 1: Solving Equations)

Since m is greater than or equal to 7.5, (D) is the answer. (Chapter 8, Lesson 6: Inequalities, Absolute Values, and Plugging In)

12. B First find the price after the 6% sales tax:

tax
price with tax

(A simpler way is just to multiply 60 by 1.06.)

Now find how much change Theo received:
change

(Chapter 7, Lesson 5: Percents)

13. A Write an equation for the first sentence.

Because none of the answer choices contain m, solve

for m in terms of r and n:

Now write an expression for what the question asks for:

Alternatively, you can substitute numbers for n, m, and r, making sure they “work,” and get a numerical answer to the question.

(Chapter 8, Lesson 1: Solving Equations)

14. D Two points on line l are (0, 0) and (10, y). Find the slope of the line:

Since , the height of the triangle is 6. Find the area:

Now find .

(Chapter 10, Lesson 4: Coordinate Geometry)

15. A Ellen travels the first 15 miles at 30 miles per hour. Find out how much time that takes:

The rest of the trip, which is miles long, she travels at an average speed of 40 miles per hour:

Add the two values together to find the total time:

(Chapter 9, Lesson 4: Rate Problems)

16. B Set up the relationship in equation form:

(Chapter 11, Lesson 4: Variation)

(Chapter 8, Lesson 2: Systems)

The answer is in terms of y alone, so find m and n in terms of y:

(Chapter 11, Lesson 6: Negative and Fractional Exponents)

19. A This question deals with similar triangles:

Shaded area = area of big triangle – area of small triangle

(Chapter 10, Lesson 6: Similar Figures)

(Chapter 10, Lesson 5: Areas and Perimeters)

20. A Set up a Venn diagram to visualize the information.

Notice that the number of sedans must equal the number of convertibles. Say the number of convertible sedans is x. If this is the number of sedans, then there must be 3x sedans in total, and of these are not convertibles. Similarly, if x is the number of convertibles, then there must be 5x convertibles altogether, and of these are not sedans. So now your diagram can look like this:

So there must be a total of cars at the dealership. The only choice that is a multiple of 7 is (A): 28.

(Chapter 9, Lesson 5: Counting Problems)

Section 4

(Chapter 10, Lesson 5: Areas and Perimeters)

Try positive integer values of a to see how many work:

There are nine integer pairs that satisfy the equation.

(Chapter 9, Lesson 3: Numerical Reasoning Problems)

3. E The ten bathrooms cost $20 each to clean:

To clean each bathroom twice would cost:

There are 30 offices, and they cost $15 each to clean:

To clean each office once and each bathroom twice will cost:

(Chapter 11, Lesson 5: Data Analysis)

4. A Remember the “difference of squares” factoring formula:

(Chapter 8, Lesson 5: Factoring)

5. A

To find the value of f (14), find all the factors of 14:

1, 2, 7, 14

There are two prime factors, 2 and 7.

To find the value of f (6), find all the factors of 6:

1, 2, 3, 6

There are two prime factors, 2 and 3.

(Chapter 11, Lesson 2: Functions)

6. D First write an equation to find the average.

Multiply by 4:

If you want a to be as large as possible, make b, c, and d as small as possible. You are told that they are all different positive integers:

(Chapter 9, Lesson 2: Mean/Median/Mode Problems)

7. B Let the radius of circle and the radius of circle . It is given that . The circumference of a circle can be found with the equation . The sum of their circumferences is 36π:

(Chapter 10, Lesson 5: Areas and Perimeters)

8. C This is a visualization problem. The six possible planes are illustrated below. Notice that the six faces of the cube “don”t count,” because each of those contains four edges of the cube.

(Chapter 10, Lesson 7: Volumes and 3-D Geometry)

(Chapter 8, Lesson 1: Solving Equations)

10. 36

There are 180° on one side of a line:

(Chapter 10, Lesson 1: Lines and Angles)

11. 5 Think simple: What”s the simplest way to turn into 2x + y? Just divide by 4!

(Chapter 8, Lesson 1: Solving Equations)

(Chapter 6, Lesson 4: Simplifying Problems)

12. 12 Just substitute and into the equation and solve for m:

(Chapter 8, Lesson 1: Solving Equations)

(Chapter 11, Lesson 2: Functions)

13. 15 Ratios such as 4:5 can also be written as . So the number of men m is 4x and the number of women w is 5x.

(Chapter 7, Lesson 4: Ratios and Proportions)

(Chapter 8, Lesson 6: Inequalities, Absolute Values, and Plugging In)

15. 25 First calculate how many grams of sucrose there are in 200 grams of a 10% mixture.

Since you will be adding x grams of sucrose, the total weight of sucrose will be 20 + x grams, and the total weight of the mixture will be 200 + x grams. Since the fraction that will be sucrose is 20%,

(Chapter 7, Lesson 5: Percents)

(Chapter 7, Lesson 4: Ratios and Proportions)

16. 24 First calculate how long the race took.

Next, find the new rate that is 25% faster:

Calculate how long the new race would take:

So she can improve her time by minutes.

(Chapter 9, Lesson 4: Rate Problems)

17. 52 Break a shape like this into recognizable four-sided figures and triangles that are easier to deal with.

The area of the rectangle on the left is . The area of the rectangle on the right is . The sum of those two areas is . The area remaining for the triangle is the difference . Set up an equation for the area of a triangle to solve for x:

To find the hypotenuse of the right triangle, set up the Pythagorean theorem and solve:

(Or just notice that it”s a 5-12-13 triangle!)

To find the perimeter of the figure, add up all of the sides:

(Chapter 10, Lesson 5: Areas and Perimeters)

(Chapter 10, Lesson 3: The Pythagorean Theorem).

18. 225 Set up a three-circle Venn diagram to visualize this information.

Fifty students study two of the three languages, so let”s say that 50 students study both Spanish and Latin. (It doesn”t matter which two languages those 50 students take; the result turns out the same.) This means that zero students study both Spanish and French, zero students study both French and Latin, and zero students study all three languages.

There are 120 Spanish students in all. There are therefore students who study Spanish alone. There are 80 French students in all, all of whom study just French, and there are 75 total Latin students including students who study only Latin.

This means that there are sophomores at Hillside High School.

(Chapter 9, Lesson 5: Counting Problems)

Section 5

1. C The clients were forced to seek more reliable investment advice, so the manager must have managed their funds badly. ineptitude = lack of skill

2. E Vartan is Armenian; he was born in Iran and educated in Lebanon and is now president of the American Brown University. He has a lot of worldly experience. perpetual = lasting forever; authoritative = showing authority; cosmopolitan = worldly

3. D They didn”t consider it in great detail, so the reading must have been without great care. verbatim = word for word; meandering = wandering; tormented = feeling anguish or pain; cursory = quick and without care; substantial = of substance, quite large

4. A If the pathogens (infectious agents) spread more quickly in close quarters, the crowding would be a problem. This would cause the disease to spread. propagation = reproduction, increase in number; squalor = horrible or dirty conditions; circulation = moving of something around from place to place; poverty = state of being poor; deterioration = wearing down; congestion = crowdedness; proximity = closeness; resilience = ability to recover from a challenge

5. E The purpose of research is to find answers to questions of interest. Therefore, the research endeavors (attempts) to determine or understand the mechanisms by which our brains do things. If the data must be turned into coherent and understandable information, it must not have been coherent to begin with, but rather just a big rush of information. enhance = make better; attenuate = reduce in amount; dearth = scarcity, lack; elucidate = make clear; deluge = huge flood

6. D The fruits mentioned in line 10 refer to the means of acquiring food and shelter, because they are described as the fruits for maintaining human life.

7. B The question is whether one can get quick returns of interest (make money) from the capital of knowledge and learning (from one”s education) (lines 13–15).

8. A The pointing of dogs is mentioned as an instinctive tendency to the performance of an action (lines 1–2).

9. E Inherited tendencies tend to show themselves in the behavior of an organism. The paragraph mentions the calf and the caterpillar as examples of organisms with instincts that show themselves in later behavior.

10. D The final paragraph begins with The best life is the one in which the creative impulses play the largest part and the possessive impulses the smallest (lines 56–58).

11. D Lines 22–26 say that the food and clothing of one man is not the food and clothing of another; if the supply is insufficient, what one man has is obtained at the expense of some other man. Therefore, food and clothing exist in finite amounts and can be used up.

12. E This section of the passage discusses matters such as good-will (line 38), science (line 31), and painting pictures or writing poems (lines 35–36) as things that are not denied to someone else when one person possesses them.

13. E This sentence discusses the possessive impulses (line 49) as distinct from the creative impulses discussed in the next sentence. The impulse of property in lines 51–52 is the desire to possess property.

14. C This statement echoes the point made in lines 71–72 that spiritual possessions cannot be taken in this way, that is, by force.

15. D Lines 58–59 say This is no new discovery and go on to cite the Gospel as a prior source expressing the same opinions as Russell”s.

16. B The author”s main point is that creativity is of higher value than possessiveness. The invention mentioned in answer choice (B) was created to make money for its inventor (a possessive and materialistic motive) but has the side effect of benefitting all of humankind.

17. A The passage discusses the perspective one Native American has on the appearance of the new superstition (line 44). It discusses how some villagers have taken to the new religion and also mentions one fellow tribe member”s attempt to convert the main character.

18. E In saying that men of the same color are like the ivory keys of one instrument where each represents all the rest, yet varies from them in pitch and quality of voice (lines 4–7), the author is saying that people of the same race possess important differences.

19. D The author describes the preacher as mouth[ing] most strangely the jangling phrases of a bigoted creed (lines 11–12), indicating that she considers him to be an intolerant person. She describes herself as having compassion (line 7) and respect (line 10), but does not attribute these qualities to the preacher.

20. B Lines 13–14 say that our tribe is one large family, where every person is related to all the others.

21. C Both the preacher and the author”s mother have become followers of the new superstition (line 44).

22. C In saying that a pugilist commented upon a recent article of mine, grossly perverting the spirit of my pen (lines 66–68), the author is saying that the pugilist distorted the author”s words in a grotesque way.

23. E The author characterizes herself as a wee child toddling in a wonder world (lines 72–73), indicating that she is in awe of the world around her. Although one might expect her to be vengeful in response to the pugilist (line 66) who grossly pervert[ed] the spirit of [her] pen (line 68), there is no indication in the paragraph that she is vengeful.

24. A The author says in lines 68–72 that still I would not forget that the pale-faced missionary and the aborigine are both God”s creatures, though small indeed in their own conceptions of Infinite Love. In other words, the author respects the missionary but believes he is small-minded.

Section 6

1. D The verb must agree with the plural subject claims. Choice (D) is most concise and correct.

2. A The original sentence is best.

3. B The participial phrase opening the sentence modifies Sartre himself, not his writing. This being the case, the phrase dangles.

(Chapter 15, Lesson 7: Dangling and Misplaced Participles)

4. C Choice (C) best follows the law of parallelism.

(Chapter 15, Lesson 3: Parallelism)

5. A The original sentence is best.

6. B Choice (B) is the most concise, logical, and complete.

(Chapter 12, Lesson 9: Write Concisely)

7. C The original phrasing contains an incomplete thought. Choice (C) is by far the most concise and direct.

(Chapter 15, Lesson 15: Coordinating Ideas)

8. E The participle having spread modifies the disease, not the doctors.

(Chapter 15, Lesson 7: Dangling and Misplaced Participles)

9. C The original phrasing contains an incomplete thought. Choice (C) is by far the most concise and direct.

(Chapter 15, Lesson 15: Coordinating Ideas)

10. D The participle singing modifies Anita, not her hoarseness. Furthermore, the participle is in the wrong form; it should be in the perfect form having sung, because only the previous singing could have contributed to her hoarseness.

(Chapter 15, Lesson 7: Dangling and Misplaced Participles)

(Chapter 15, Lesson 9: Tricky Tenses)

11. A The original sentence is best.

12. A The word quick is an adjective and can thus modify only a noun. But since it modifies the verb turned, the adverb quickly is needed here.

(Chapter 15, Lesson 12: Other Modifier Problems)

13. B This sentence violates the law of parallelism. If she is known for her initiative, she should also be known for devoting her own time.

(Chapter 15, Lesson 3: Parallelism)

14. C Since the Medieval era is long past, its beginning is “completed” or, in grammar terms, “perfect.” So this phrase should be the “perfect” form of the infinitive: to have begun.

(Chapter 15, Lesson 9: Tricky Tenses)

15. B The word neither is almost always part of the phrase neither of … or neither A nor B. So choice (B) should read nor even.

(Chapter 15, Lesson 10: Idiom Errors)

16. D The word less is used to compare only quantities that can”t be counted. If the quantities are countable, as accidents are, the word should be fewer.

(Chapter 15, Lesson 4: Comparison Problems)

17. B To convey the proper sequence of events, the perfect tense is required: had spent.

(Chapter 15, Lesson 9: Tricky Tenses)

18. A The subject of the verb has is the plural noun newspapers. (The sentence is “inverted,” because the subject follows the verb.) The proper form of the verb, then, is have.

(Chapter 15, Lesson 1: Subject-Verb Disagreement)

(Chapter 15, Lesson 2: Trimming Sentences)

19. B The original sentence has a “comma splice” that incorrectly joins two sentences with only a comma. A better phrasing is dream that led.

(Chapter 15, Lesson 15: Coordinating Ideas)

20. C The subject of the verb is the singular noun movement, so the proper verb form is has led.

(Chapter 15, Lesson 1: Subject-Verb Disagreement)

(Chapter 15, Lesson 2: Trimming Sentences)

21. E The sentence is correct as written.

22. D This is a prepositional phrase, so the pronoun is the object of the preposition and should be in the objective case. The correct phrasing is for Maria and me.

(Chapter 15, Lesson 6: Pronoun Case)

23. A The word successive means consecutive, so it does not make sense in this context. The right word is successful.

(Chapter 15, Lesson 11: Diction Errors)

24. E The sentence is correct as written.

25. C The word underneath means that it is physically below something else. It should be changed to under.

(Chapter 15, Lesson 10: Idiom Errors)

26. E The sentence is correct as written.

27. B The subject of the verb were is arrogance, which is singular. It should instead be was.

(Chapter 15, Lesson 1: Subject-Verb Disagreement)

28. B The sentence mentions there are numerous strains of the bacteria, which means that more should instead be most.

(Chapter 15, Lesson 4: Comparison Problems)

29. C The subject company is singular. Therefore, they should instead be it.

(Chapter 15, Lesson 5: Pronoun-Antecedent Disagreement)

30. D Choice (D) is most consistent, logical, and concise.

31. A Choice (A) is most logical.

(Chapter 12, Lesson 7: Write Logically)

32. B The first paragraph ends with the description of an idea. The second paragraph begins with an illustration of how students experience this idea in their daily lives and then goes on to explain how it can help them get through their brain freezes. Choice (B) is the best introduction to the paragraph, because it explains that a student using the phenomenon can improve his or her studies.

(Chapter 12, Lesson 7: Write Logically)

33. C The sentence begins using the pronoun you, so that usage should be maintained throughout the sentence. Choice (D) is incorrect because a person has only one brain.

(Chapter 15, Lesson 5: Pronoun-Antecedent Disagreement)

34. E Sentence 11 concludes a discussion of Isaac Asimov”s “eureka” experience. The additional sentence expands upon that idea, relating it back to the lives of students.

(Chapter 12, Lesson 7: Write Logically)

35. C Choice (C) is the most concise and logical revision.

(Chapter 12, Lesson 7: Write Logically)

(Chapter 12, Lesson 9: Write Concisely)

Section 7

1. B Set up a ratio to solve this problem:

(Chapter 7, Lesson 4: Ratios and Proportions)

(Chapter 8, Lesson 3: Working with Exponentials)

3. A The sum of a, b, and 18 is 6 greater than the sum of a, b, and 12. Since there are three terms in the group, it follows that the average of a, b, and 18 would be greater than the average of a, b, and 12.

(Chapter 9, Lesson 2: Mean/Median/Mode Problems)

4. B If you have the patience, you can write out a quick calendar for yourself to track the days:

Or you can use the simple fact that successive Tuesdays (like any other days) are always 7 days apart. Therefore, if the 1st of the month is a Tuesday, so are the 8th, the 15th, the 22nd, and the 29th. Therefore, the 30th is a Wednesday and the 31st is a Thursday.

(Chapter 9, Lesson 3: Numerical Reasoning Problems)

Since n must be an integer, n can be 1, 2, 3, 4, or 5.

(Chapter 8, Lesson 6: Inequalities, Absolute Values, and Plugging In)

6. D First find the value of y: y% of 50 is 32.

What is 200% of 64?

(Chapter 7, Lesson 5: Percents)

(Chapter 11, Lesson 2: Functions)

8. C The slope of the line is –¾, so use the slope equation and the coordinates of point A (0, 12) to find the coordinates of point B (x, 0):

The base of the triangle is 16, and its height is 12.

(Chapter 10, Lesson 4: Coordinate Geometry)

9. A Find the sum of each repetition of the pattern:

Next, determine how many times the pattern repeats in the first 25 terms: with a remainder of 1.

Multiply the sum of the pattern by 8 to obtain the sum of the first 24 terms:

The 25th term is 1, which makes the sum .

(Chapter 11, Lesson 1: Sequences)

10. D The ratio of white marbles to blue marbles is 4 to b. The probability of randomly selecting a white marble from the jar is ¼. This means that one out of every four marbles in the jar is white and three out of every four marbles are blue. If there are four white marbles, then there are blue marbles.

(Chapter 7, Lesson 4: Ratios and Proportions)

11. B Area = ½(base)(height)

The base and the height are both integers. Find all the “factor pairs” of 20: 1, 20; 2, 10; and 4, 5

Plug each pair into the Pythagorean theorem to find the least possible length of the hypotenuse:

is the shortest possible hypotenuse.

(Chapter 10, Lesson 5: Areas and Perimeters)

(Chapter 10, Lesson 3: The Pythagorean Theorem)

This means that y is a negative decimal fraction. Answer choices (A), (C), and (E) will all be negative numbers. Answer choices (B) and (D) are positive numbers. When you raise a simple fraction to a positive number larger than 1, it gets smaller. , which makes (B) the greatest value. Pick a value like and see.

(Chapter 9, Lesson 3: Numerical Reasoning Problems)

13. E Any statement of the form “If A is true, then B is true” is logically equivalent to “If B is not true, then A is not true.” Try this with some common-sense examples of such statements. For instance, saying “If I am under 16 years old, then I am not allowed to drive” is the same as saying“If I am allowed to drive, then I must not be under 16 years old.” The statement in (E) is logically equivalent to the original.

(Chapter 6, Lesson 7: Thinking Logically)

14. E If each bus contained only the minimum number of students, the buses would accommodate students. But since you have 200 students to accommodate, you have 20 more students to place. To maximize the number of 40-student buses, place 10 more students in two of the buses. Therefore, a maximum of two buses can have 40 students.

(Chapter 9, Lesson 3: Numerical Reasoning Problems)

15. D The volume of a cylinder is equal to πr2h. Let”s say that the radius of cylinder A is a and the radius of cylinder B is b. Since the height of cylinder B is twice the height of cylinder A, if the height of cylinder A is h, then the height of cylinder B is 2h. The volume of

(Chapter 10, Lesson 7: Volumes and 3-D Geometry)

16. C The key is to find a pattern among the many possible solutions. Pick some values for x to see if you can see a pattern. For instance, if , then the garden looks like this:

In this case . But if x = 4, the garden looks like this:

And here, . You might notice that the value of w has increased by 4. Does this pattern continue? Let”s try to check:

Sure enough, , and it seems that the pattern continues and w is always a multiple of 4. Only choice (C), 40, is a multiple of 4, so that must be the correct answer.

(Chapter 6, Lesson 3: Finding Patterns)

Section 8

1. B A reputable scientist is well known and well respected. Saying the evidence is-------at best indicates that there is not much evidence at all. It must be flimsy. Reputable scientists would not likely admit that a phenomenon exists if the evidence is weak. meager = scanty, deficient; regret= feel bad about an action, wish it hadn”t happened; paltry = lacking worth

2. D The concept that the Earth is round is now accepted as an inarguable truth. It can be inferred that it was at some point a fact that was thought to be wrong. incontrovertible = cannot be questioned; mellifluous = smooth flowing; dubious = doubtful

3. B A profound break of a political party or religion into factions is a schism. (The Latin word schisma = split.) unanimity = full agreement; schism = division into factions; caucus = meeting of party members; commemoration = event that honors something or someone; prognostication = prediction

4. C As the father of the American public school system, Horace Mann would pressure or push the Massachusetts legislature to institute a system for ensuring or guaranteeing universal access to eduction. petitioned = requested, lobbied for; vouchsafing = conceding, granting

5. A Since the light from most stars takes millions of years to reach us, it is plausible to imagine that by the time we see the light the star might actually no longer be there. This would make the present existence of these stars questionable. debatable = disputable; methodical = systematic;indecorous = not proper; imperious = acting as if one is superior to another; profuse = abundant

6. D The although establishes a contrast. Something that makes any potentially offensive matters seem less objectionable is, by definition, a euphemism. The first blank should therefore be a word that contrasts with euphemism, like straightforward. anachronism = something out of place in time; intolerant = unable to put up with something; laudation = extreme praise; clandestine = secret, hidden; candid = honest, straightforward; euphemism = the substitution of an inoffensive term for an offensive one; forthright = honest; coercion = pressure on someone to act

7. C The English gentleman tried to teach his son Greek and Latin without punishment, … rewarding his son with cherries and biscuits (lines 5–11).

8. A In saying that marks, grades, and diplomas … must be made reinforcing for other reasons (lines 16–19), the author is saying that such things will not reinforce behavior by themselves but must be made to represent something more meaningful.

9. B The passage says that how honors and medals derive their power from prestige or esteem is what varies between cultures and epochs (lines 20–23). When Oscar Wilde got a “first in Mods” in 1876, he was the talk of the town. But the contemporary student graduating summa cum laude is less widely acclaimed (lines 33–34).

10. A The story follows the statement that how honors and medals derive their power from prestige or esteem is what varies between cultures and epochs. Therefore, the story is intended to illustrate that fact.

11. C Statement I is supported by lines 37–39, which say that certain kinds of reinforcements (like food) are not always easily arranged. Statement II is supported by line 43: We cannot all get prizes. The selection does not mention anything about rewards encouraging only superficial learning.

12. D In lines 58–60, the passage says that grades are almost always given long after the student has stopped behaving as a student. It then goes on to discuss how such contingencies are weak (lines 60–61).

13. E The paragraph as a whole discusses the need for teachers to address the issues of whether, when, and how to punish or reward student behavior, so it is about teacher-student interactions.

14. B Kohn and Deci are mentioned as examples of experts who believe that reward is often just as harmful as punishment, if not more so (lines 87–89).

15. D The second paragraph of Passage 2 goes on to argue that those who are doing a task without a reward continue to perform the task because they see it as being “fun,” whereas those who do it for a reward stop playing because they are no longer being paid to continue. The activity”s sole value comes from the payment they get for it, not from the enjoyment they get from participating.

16. D We are told that Deci concluded that the subjects who were paid probably construed (interpreted) the task as being manipulative (lines 105–106). In order to draw such conclusions, the subjects would have to make inferences about the motivations of the experimenter.

17. A The author follows that statement with it would be a mistake to use these few experiments to generalize that all rewards are bad (lines 130–132). These statements caution against an overly simplistic theory about the effectiveness of rewards.

18. B Deci”s opinion is that the introduction of a reward system changes things for the worse. He would see the description of the problems mentioned in line 37 as presumptuous because they presume that the rewards actually have a positive effect and incomplete because they do not mention all of the problems that he sees in reward systems.

19. D Both authors agree that positive feedback is a more effective teaching mechanism than negative feedback. Passage 1 mentions the need of good educators to teach … without punishment (lines 6–7) and mentions the negative by-products of aversive control (control by punishment) (lines 35–36). Passage 2 mentions that most educators and psychologists agree that reward is always better than punishment (lines 84–85), and since the writer goes on to criticize even reward systems, he implies that punishment is most certainly a bad teaching technique.

Section 9

1. C The word whose should refer to Alvin Ailey, but the way the sentence is constructed, it is referring to Alvin Ailey”s works. Answer choice (C) corrects this error in the most concise and logical fashion.

(Chapter 15, Lesson 2: Trimming Sentences)

2. B When a participle is used to indicate an action that is completed before another action, it should be perfect. Getting this far should instead be Having gotten.

(Chapter 15, Lesson 9: Tricky Tenses)

3. C The sentence is improperly describing Rachel”s irritation as being even-tempered. In reality, it should be Rachel who is even-tempered. Answer choice (C) corrects this error.

(Chapter 15, Lesson 7: Dangling and Misplaced Participles)

4. B The original sentence contains an illogical comparison. Alberta”s salary cannot logically be compared to the other agents, but rather must be compared to the salaries of the other agents. The only choice that makes a logical and idiomatic comparison is choice B.

(Chapter 15, Lesson 4: Comparison Problems)

5. C The word elicit means to call forth or draw out. The word should be illicit, which means unlawful.

(Chapter 15, Lesson 11: Diction Errors)

6. D The paraders were not watching from the balcony. The sentence needs to be changed so that the subjects represented by the final pronoun us are the ones watching from the balcony.

(Chapter 15, Lesson 7: Dangling and Misplaced Participles)

7. E The sentence contains two past tense verbs, and one event was completed before the other. The tents were set up before they arrived. So set up needs to be in the past perfect tense—had set up.

(Chapter 15, Lesson 9: Tricky Tenses)

8. C The subject neither Matt nor David is singular, because any neither A nor B noun phrase takes the same number as B, which in this case is the singular David. Therefore, the verb should be conjugated for a singular subject: neither Matt nor David is permitted.

(Chapter 15, Lesson 1: Subject-Verb Disagreement)

9. B When using not only A but also B, the words or phrases that replace A and B must be parallel. It should be replaced by not only with working but also with wanting.

(Chapter 15, Lesson 3: Parallelism)

10. C To correct this sentence, the word that should be replaced with who, since Roberto is a person.

(Chapter 15, Lesson 5: Pronoun-Antecedent Disagreement)

11. D Answer choice (D) connects the two clauses most effectively.

(Chapter 15, Lesson 15: Coordinating Ideas)

12. E When reading this sentence you should ask yourself: “who was forced to live apart from his family?” The answer to that question, St. Pierre, is what should immediately follow the comma after informers.

(Chapter 15, Lesson 7: Dangling and Misplaced Participles)

13. B The gerund form, forming, is not correct and needs to be changed to past tense formed. Choice (B) works best.

(Chapter 15, Lesson 9: Tricky Tenses)

14. D What follows the linking verb is must be a noun phrase representing the most challenging aspect, not an independent clause, as in the original. Choice (D) works best.

(Chapter 15, Lesson 15: Coordinating Ideas)